O que pode ser dito sobre a soma das séries?

Aug 15 2020

Deixei $\{a_n \}_{n \geq 1}$ ser uma sequência de inteiros diferentes de zero satisfazendo

EU. $|a_n| \lt |a_{n+1}|,$ para todos $ n \geq 1$

II. $a_n$ divide $a_{n+1},$ para todos $n \geq 1$ e

III. cada inteiro é um divisor de algum$a_n.$

Então $\displaystyle\sum\limits_{n=1}^{\infty} \frac {1} {a_n}$ é

(a) absolutamente convergente e sua soma é um número racional.

(b) absolutamente convergente e sua soma é um número irracional.

(c) absolutamente convergente e sua soma é um número positivo.

(d) nenhuma das opções acima.

Minha tentativa $:$ É fácil ver que $\displaystyle\sum\limits_{n=1}^{\infty} \frac {1} {a_n}$é absolutamente convergente. Deixei$a_{k+1} = m_k\ a_{k},$ para $k \geq 1.$ Por (I) segue-se que $|m_k| \geq 2,$ para todos $k \geq 1.$ Então nós temos

\begin{align*} \sum\limits_{n=1}^{\infty} \frac {1} {|a_n|} & = \frac {1} {|a_1|} + \frac {1} {|a_2|} + \frac {1} {|a_3|} + \cdots \\ & = \frac {1} {|a_1|} + \frac {1} {|m_1|\ |a_1|} + \frac {1} {|m_2|\ |a_2|} + \cdots \\ & = \frac {1} {|a_1|} + \frac {1} {|m_1|\ |a_1|} + \frac {1} {|m_1|\ |m_2|\ |a_2|} + \cdots \\ & \leq \frac {1} {|a_1|} \left ( 1 + \frac {1} {2} + \frac {1} {2^2} + \cdots \right ) \\ & = \frac {2} {|a_1|} < \infty \end{align*}

então $\displaystyle\sum\limits_{n=1}^{\infty} \frac {1} {a_n}$é absolutamente convergente. Claramente (a) é falso porque podemos pegar$a_n = n!,$ para todos $n \geq 1.$ Então a soma é $e-1,$o que é claramente irracional. Posso muito bem pegar$a_n = -n!,$ para todos $n \geq 1.$ O que faz a soma $1-e,$uma quantidade negativa. Portanto, (c) também é falso. Mas como posso concluir que se a soma é sempre irracional ou não? Qualquer ajuda a este respeito será muito apreciada.

Desde já, obrigado.

Respostas

6 StephenMontgomery-Smith Aug 15 2020 at 19:45

Deixei $$ R_m = \sum_{n > m} \frac 1{a_n} .$$

Lema: $0 < \displaystyle |R_m| < \frac1{|a_m|} .$

Prova: Deixe $r \ne 2$ ser um número primo que não é um fator de $a_{m}$. Então existe$m' > m$ de tal modo que $r | a_{m'}$. Assim para$n > m$ temos $|a_n| \ge 2^{n-m} |a_m|$, e para $n \ge m'$ temos $|a_n| \ge r 2^{n-m-1} |a_m|$. portanto$$|R_m| \le \sum_{n > m} \left|\frac {1}{a_n} \right| \le \frac1{|a_{m}|}\left(\sum_{n=m+1}^{m'-1} 2^{m-n} + \frac 2 r \sum_{n=m'}^\infty 2^{m-n}\right) < \frac1{|a_m|}.$$ Para o limite inferior, $R_m = \frac1{a_m} + R_{m+1}$, então $$|R_m| = \frac1{|a_m|} - |R_{m+1}| \ge \frac1{|a_m|} - \frac1{|a_{m+1}|} > 0 .$$

$\square$

Suponha $$S = \sum_n \frac1{a_n} = \frac pq $$ Onde $p,q \ne 0$ são inteiros.

Para alguns $m$, temos $q | a_m$. Então$a_m S = \text{integer} + a_m R_m$é um número inteiro. Do Lema, vemos que$0 < |a_m R_m| < 1$. então$a_m R_m$ não pode ser um inteiro, e então $S$ não pode ser racional.